2001 제14회 한국수학올림피아드 최종시험

1일: 1번문제~3번문제, 2001년 4월 14일 4시간 30분.
2일: 4번문제~6번문제, 2001년 4월 15일 4시간 30분.

최우수상 (2명, 가나다순)

김명섭(경기과학고 2) 이해강(서울과학고 1)

우수상 (37명, 가나다순)

곽승재(민족사관고 2) 권수현(부산과학고 3) 권영대(전북과학고 2) 권익재(경북과학고 3) 김대원(민족사관고 3) 김동현(서울과학고 3) 김동환(부산용인고 3) 김린기(유성고 2) 김석원(서울과학고 3) 김재우(서울과학고 2) 류동관(안양고 2) 서상현(경기과학고 1) 안주용(인천과학고 3) 안준규(경기과학고 2) 연홍윤(한성과학고 1) 오찬우(서울과학고 1) 은동재(서울과학고 2) 이관석(경기과학고 3) 이도현(서울과학고 2) 이승주(대전과학고 1) 이종훈(경기과학고 2) 이준노(서울과학고 3) 이준영(충남과학고 3) 이준영(휘문고 1) 이중범(대전과학고 2) 이지훈(포항제철고 3) 장영준(서울과학고 3) 정용욱(대구과학고 3) 조승연(서울과학고 1) 조재현(경기과학고 2) 하준수(여의도고 3) 한필구(반포고 3) 허예솔(경기과학고 1) 현윤석(안양고 3) 홍석범(서울과학고 2) 홍세린(서울과학고 1) 황지은(서울과학고 1)

문제출처 수상자출처

GD Star Rating
loading...

홀수인 소수 $p$에 대하여, 다음 두 조건을 만족시키는 함수 $f:\mathbb Z\to\mathbb Z$를 모두 구하여라. 단, $\mathbb Z$는 정수 전체의 집합이다.
(1) $m\equiv n\pmod p$이면 $f(m)=f(n)$이고,
(2) 임의의 정수 $m$, $n$에 대하여, $f(mn)=f(m)f(n)$이다.

볼록사각형 $O_1O_2O_3O_4$의 내부에 고정된 점 $P$가 있다. 각 $i = 1,2,3,4$에 대하여, 점 $P$를 지나고 두 반직선 $\overrightarrow{O_iO_{i+1}}, \overrightarrow{O_iO_{i-1}}$과 각각 서로 다 른 두 점 $A_i$, $B_i$에서 만나는 직선 $\ell$을 생각하자. 두 선분 $PA_i$, $PB_i$의 길이의 곱 $\overline{PA_i}\cdot \overline{PB_i}$가 최소가 되도록 하는 직선 $\ell$을 $\ell_i$라고 할 때,
$\ell_1 =\ell_3$, $\ell_2 =\ell_4$이면 사각형 $O_1O_2O_3O_4$가 평행사변형임을 보여라. 단, $\vec{XY}$는 점 $X$에서 시작하여 점 $Y$로 향하는 반직선이고, $O_0 =O_4$, $O_5 =
O_1$이다.

등식 $x_1^2+x_2^2+\cdots+x_n^2=1$, $y_1^2+y_2^2+\cdots+y_n^2=1$을 만족시키는 임의의 실수 $x_1,x_2,\ldots,x_n$과 $y_1,y_2,\ldots,y_n$에 대하여 \[ (x_1y_2-x_2y_1)^2\le 2\left\lvert 1-\sum_{k=1}^n x_ky_k\right\rvert\]임을 보여라. 또, 등호가 성립할 조건을 구하여라.

주어진 양의 정수 $n$과 $N$에 대하여, 다음 두 조건을 만족시키는 정수 계수의 다항식 $f(x)=a_0+a_1x+\cdots+a_nx^n$을 생각하자.
(1) $\lvert a_i\rvert \le N$, $j=0,1,\ldots,n$,
(2) $\{ j | a_j=N\}$의 원소의 개수는 $2$ 이하이다.
이러한 다항식 전체의 집합 $P_n$에 대하여, 집합 $\{ f(2N) | f(x)\in P_n\}$의 원소의 개수를 구하여라.

$\triangle ABC$의 변 $BC$ 위의 점 $D$에 대하여, $\triangle ABD$의 내심과 $\triangle ABC$의 외심이 점 $O$에서 일치한다고 하자. $\triangle AOC$의 외접원을 $O’$이라고 하고, 점 $O’$의 두 점 $A$와 $C$에서의 두 접선의 교점을 $P$, 선분 $AD$와 $CO$의 교점을 $Q$라고 하자. 원 $O’$의 점 $O$에서의 접선과 직선 $PQ$가 점 $X$에서 만날때, $\overline{XO}=\overline{XD}$임을 보여라. 단, $\angle B<45^\circ$이다.

양의 정수 $n\ge 5$에 대하여, 정수 $a_1,b_1,a_2,b_2,\ldots,a_n,b_n$이 다음의 두 조건을 만족시킨다고 하자.
(1) 모든 $i = 1,2,\ldots,n$에 대하여 순서쌍 $(a_i,b_i)$는 모두 서로 다르고,
(2) $\lvert a_1b_2 – a_2b_1\rvert =\cdots = \lvert a_{n-1}b_n- a_nb_{n-1}\rvert = \lvert a_nb_1 – a_1b_n\rvert = 1$이다.
이 때, $1 < \lvert i -j \rvert < n – 1 $이고 $\lvert a_i b_j-a_jb_i\rvert = 1 $을 만족시키는 $i$, $j$가 존재함을 보여라.

답글 남기기